A box of Lucky Charms costs $3.29 and you have a coupon that will save you 75 cents off 2 containers. What is the final price if you buy 2?​

Answers

Answer 1
The final price of the lucky charms is $2.54

Related Questions

Please help I’ll give brainliest

Answers

Answer:

2 is the base

Step-by-step explanation:

2^3

2 is the base and 3 is the exponent

The answer is the base is 2 (first option)

Condense the expression into a
single logarithm and simplify.
2log10x + + log105

Answers

Answer:

log10( 5x^2)

Step-by-step explanation:

2 log10 (x) + log10 (5)

we know that  c loga (b) = log a (b^c)

log10 x^2 + log10 (5)

We also know that logc a + logc b = logc (ab)

log10 (x^2* 5)

log10( 5x^2)

Pls help. It’s quite late <3

Answers

Answer:

a) 120

Step-by-step explanation:

B is twice the size of A

pleaae help me solve this 61/2×(8/9÷13/18)+(3/4) of 31/5​

Answers

Answer:

[tex]10 \frac{2}{5}[/tex]

Step-by-step explanation:

Using BODMAS

[tex]6\frac{1}{2} \times (\frac{8}{9} \div\frac{13}{18}) + ( \frac{3}{4} )\ of \ 3\frac{1}{5} \ \ \ \ \ \ \ \ \ \ \ \ \ \ \ [ \ solving \ expression \ inside \ bracket \ ]\\\\\frac{13}{2} \times (\frac{8}{9} \times \frac{18}{13}) + (\frac{3}{4}) \ of \ \frac{16}{5} \ \ \ \ \ \ \ \ \ \ \ \ \ \ \ [ \ \frac{a}{b} \div \frac{c}{d} = \frac{a}{b} \times \frac{d}{c} \ ]\\\\\frac{13}{2} \times (\frac{16}{13}) + (\frac{3}{4}) \ of \frac{16}{5} \ \ \ \ \ \ \ \ \ \ \ \ \ \ \ \ \ \ \ \ \ [ \ solving \ of \ ] \\\\[/tex]

[tex]\frac{13}{2} \times (\frac{16}{13} ) + (\frac{3}{4} \times \frac{16}{5} )\\\\\frac{13}{2} \times (\frac{16}{13} ) + \frac{12}{5} \ \ \ \ \ \ \ \ \ \ \ \ \ \ \ \ \ \ \ \ [\ solving \ \times \ expressions \ ] \\\\(\frac{13}{2} \times \frac{16}{13}) + \frac{12}{5}\\\\8 + \frac{12}{5}\\\\\frac{40 + 12}{5}\\\\\frac{52}{5}\\\\10\frac{2}{5}[/tex]

Which verbal expression shows 4 ÷ n?

Your answer:

n divided by 4


the quotient of 4 and n


the product of 4 and n


4 decreased by n

Answers

Answer:

'the quotient of 4 and n ' is the correct answer

Students are asked to estimate the number of gumballs in a jar. Sam says there are 228 gumballs. In actuality, there are 240 gumballs. What is the percent error

Answers

Answer:

5%

Step-by-step explanation:

Percent error = (actual - estimated) / actual x 100

(240 - 228) / 240 x 100 = 5%

15
9
determine the value
coso

Answers

Answer:

36.87°

Step-by-step explanation:

Given the right angle triangle :

To obtain the value of Cosθ ; we use the trigonometric relation :

Cosθ = Adjacent / Hypotenus

The adjacent angle isn't given :

Opposite = 9 ; hypotenus = 15

Adjacent = √(hypotenus ² + opposite ²)

Adjacent = √(15² - 9²)

Adjacent = √(225 -81)

Adjacent = √144 = 12

Hence,

Cos θ = 12/15

θ = Cos^-1(12/15)

θ = 36.87°

What is the range of the given function?

{(–2, 0), (–4, –3), (2, –9), (0, 5), (–5, 7)}

A {x | x = –5, –4, –2, 0, 2}
B {y | y = –9, –3, 0, 5, 7}
C {x | x = –9, –5, –4, –3, –2, 0, 2, 5, 7}
D {y | y = –9, –5, –4, –3, –2, 0, 2, 5, 7}

Answers

[tex]\underline \bold{ \: \: \: \: \: \: \: \: \: \: \: \: \: \: \: \: \: \: \: \: \: \: \: \: \: \: \: \: \: \: \: \: \: \: \: \: \: \: \: \: \: \: \: \: \: \: \: \: \: \: \: \: \: \: \: \: \: \: \: \: \: \: \: \: \: \: \: \: \: \: \: \: \: \: \: \: \: \: \: \: \: \: }[/tex]

[tex]\huge\underline{\sf{\red{Problem:}}}[/tex]

What is the range of the given function?(–2, 0), (–4, –3), (2, –9), (0, 5), (–5, 7)

[tex]\huge\underline{\sf{\red{Choices:}}}[/tex]

A {x | x = –5, –4, –2, 0, 2}

B {y | y = –9, –3, 0, 5, 7}

C {x | x = –9,–5,–4,–3,–2,0,2,5,7}

D {y | y = –9,–5,–4,–3,–2,0,2,5,7}

[tex]\huge\underline{\sf{\red{Answer:}}}[/tex]

[tex] \quad \quad \underline{ \boxed{\sf{ \red{ B.)\: {y | y = –9, –3, 0, 5, 7} }} }}[/tex]

What is range of a function?The range of a function is the complete set of all possible resulting values of the dependent variable (y, usually), after we have substituted the domain.

The definition means:

The range is the resulting y-values we get after substituting all the possible x-values.

How to find the range?

The range of a function is the spread of possible y-values (minimum y-value to maximum y-value)Substitute different x-values into the expression for y to see what is happening. (Ask yourself: Is y always positive? Always negative? Or maybe not equal to certain values?)Make sure you look for minimum and maximum values of y.

[tex]\underline \bold{ \: \: \: \: \: \: \: \: \: \: \: \: \: \: \: \: \: \: \: \: \: \: \: \: \: \: \: \: \: \: \: \: \: \: \: \: \: \: \: \: \: \: \: \: \: \: \: \: \: \: \: \: \: \: \: \: \: \: \: \: \: \: \: \: \: \: \: \: \: \: \: \: \: \: \: \: \: \: \: \: \: \: }[/tex]

#CarryOnLearning

[tex]\sf{\red{✍︎ C.Rose❀}}[/tex]

Neil is buying steak for a cookout on Saturday. Steak is on sale for $9.62 per pound. If he buys 7.5 pounds of steak, how much money does he spend?

Answers

Answer:

$72.15

Step-by-step explanation:

x= pounds of steak

$[tex]9.62x[/tex] = money spent on steaks

If Neil buys 7.5 pounds of steak then x =7.5

$9.62(7.5) = $72.15

Answer:

$72.15

Step-by-step explanation:

1 pound = $9.62

7.5 pounds = $9.62 × 7.5

= $72.15

if 2x + 3y = 12 and xy = 6, find the value of 8x^3 + 27y^3​

Answers

Answer:

The value of [tex]8\cdot x^{3} + 27\cdot y^{3}[/tex] is 432.

Step-by-step explanation:

Let be the following system of equations:

[tex]2\cdot x + 3\cdot y = 12[/tex] (1)

[tex]x\cdot y = 6[/tex] (2)

Then, we solve both for [tex]x[/tex] and [tex]y[/tex]:

From (1):

[tex]2\cdot x + 3\cdot y = 12[/tex]

[tex]2\cdot x = 12- 3\cdot y[/tex]

[tex]x = 6 - \frac{3}{2}\cdot y[/tex]

(1) in (2):

[tex]\left(6-\frac{3}{2}\cdot y \right)\cdot y = 6[/tex]

[tex]6\cdot y-\frac{3}{2}\cdot y^{2} = 6[/tex]

[tex]\frac{3}{2}\cdot y^{2}-6\cdot y + 6 = 0[/tex]

The roots of the polynomial are determined by the Quadratic Formula:

[tex]y_{1} = y_{2} = 2[/tex]

By (1):

[tex]x = 6 - \frac{3}{2}\cdot (2)[/tex]

[tex]x = 3[/tex]

If we know that [tex]x = 3[/tex] and [tex]y = 2[/tex], then the final value is:

[tex]z = 8\cdot x^{3}+27\cdot y^{3}[/tex]

[tex]z = 8\cdot 3^{3}+27\cdot 2^{3}[/tex]

[tex]z = 432[/tex]

The value of [tex]8\cdot x^{3} + 27\cdot y^{3}[/tex] is 432.

Pls help will mark brainliest and 30 points

Answers

1. Answer is option 3

2. Answer is option 3

Answer:

for #1 it's the 3rd one and for #2 it's the 4th one (2.8 meters)

The Jenkins family's monthly budget is
shown in the circle graph. The family has a
monthly income of $4,800. How much
money do they spend on transportation
each month?
Emergency fund
5%
Transportation
5%
Housing
30%
Medical
22%
Food
15%
Clothing
6%
Entertainment
7%
Savings
10%​

Answers

Answer:

$240

Step-by-step explanation:

They spend 5% of their monthly income on transportation, so you want to work out 5% of 4800.

An easy trick with 5% is to divide by 10 (drop the ending 0) and then divide by 2. This would be 4800/10/2, or 480/2, which gives 240.

**This content involves using percentages which you may wish to revise.

Please help don’t understand at all.

Answers

Answer:

tringlae mixed witha football goal take the fool goal out and go from there

Step-by-step explanation:

Hi can anyone help me with this Q ? It’s for my final. I would appreciate it a lot !!
-
The angle of elevation from a boat to the top of a 125ft hotel is 22°
How far is the boat from the base of the hotel ? ( round to the nearest TENTH )

Answers

Answer: B

Step-by-step explanation:

Bc said so

(a+b)(a-b)=? Đây là hằng đẳng thức nào?

Answers

Answer:

không

Step-by-step explanation:

Answer:

(a + b)(a - b) = a² - b²

Step-by-step explanation:

(a + b)(a - b) = a (a - b ) + b (a - b )

                 = a²  - ab + ba  - b²             [ ab = ba ]

                 = a² - ab + ab - b²               [ ab - ab = 0 ]

                 = a² - b²

Use a calculator to find the r-value of these data. Round the value to three
decimal places


Answers

Answer:

-.985

Step-by-step explanation:

-.985 is the answer

give your answer in detail plzz ​

Answers

Answer:

60% did not germinate.

Step-by-step explanation:

100% = 5

how many percent are 2 of of these 5 ?

2 = 2/5 × 5 = 2/5 × 100% = 40%

all the other seeds are then the rest of 100%.

100% - 40% = 60%

=>

60% of the seeds did not germinate.

but 40% did.

What is the surface area of LMNOPQRS? If necessary, round your answer to the nearest tenth.

Answers

Answer:

5

Step-by-step explanation:

If the first three Fibonacci numbers are given as x, = 1, X2 = 1 and x3 = 2,
what is the value of n for which X, + Xp = 55?

Answers

Can you please help me with this question

Answer:

n=8

Step-by-step explanation:

Simplify the expression....​

Answers

Answer:

−3x^2+2x /x−2

Step-by-step explanation:

4x−9x^3/ 3x^2−4x−4

=  −9x^3+4x /3x^2−4x−4

=  x(−3x+2)(3x+2) /(3x+2)(x−2)

=  −3x^2+2x /x−2

HELP!!!
f(x) =x^2. What is g(x)?

Answers

Answer:

C. g(x) = 4x²

Step-by-step explanation:

we have one specific point on g(x) : (1,4)

so, for x=1 the functional value (y) has to be 4.

A. (4×1)² = 4² = 16 and not 4. => wrong

B. 1/4 × 1² = 1/4 and not 4. => wrong

C. 4×1² = 4 and that IS 4 => correct

D. 16×1² = 16 and not 4 => wrong

Which fraction converts to a repeating decimal number?
CA.
1
12
B.
718
C.
127
27
D.
E.
6
10
Reset

Answers

Answer: A.

Step-by-step explanation:

Data: Fraction that turning into a repeating decimal number=x

Only step: Divide all the fractions, 1/12, 7/8, 14/25, 17/20, 6/10

Explanation: The only way to find which fraction turns into a repeating decimal is by dividing all the fractions, this can be done in any order but for this problem, lets start with 1/12 which, when divided, turns into 0.083... which is a repeating decimal

With that being said, the answer would be A.(1/12)

I hope this helps(Mark brainliest if you'd like to)

The ordered pairs (6,8), (9, 12), (x, 20), (18, y) represent a proportional relationship. Find the value of X and the value of Y. Select the two correct answers.

Answers

Answer:

x=15 y=24

Step-by-step explanation:

This is because the rule is x goes up 3 y goes up 4

(9,12)->(x,20)

it skips one, so instead of adding 3 we add six

x=15

(15,20)->(18,y)

you follow the rule and 4.

Y=24

(4p _ 2k)(3)
in distributive property

Answers

Assuming that _ is meant to be a minus sign,
12p - 6k
3(4p) - 3(2k)

Answer:

12 p - 6k

Step-by-step explanation:

Let us assume that _ is meant to be minus sign.

( 4 p - 2k ) ( 3)

use the distributive property

3 × 4p - 3 × 2k

12 p - 6 k

Please hurry i want the answer of this question please

Answers

[tex]\displaystyle\bf 1200=12*100=3*4*(2*5)^2=3*2^2*2^2*5^2=2^4*3^1*5^2 \\\\Answer: \boxed{ A)\quad a=4 \quad ; \quad b=1 \quad ; \quad c=2}[/tex]

Please help I wanna pass

Answers

Answer:

Everything is correct except for the second one and the ones you checkmarked.

Step-by-step explanation:

find the volume of this cone. round to the nearest tenth. l=10 r-6

Answers

Answer:

376.8

Step-by-step explanation:

Formula Volume Cone = πr² * h/3

Volume = 3.14 * 36 * 3 1/3

Volume ≈ 376.8

If my answer is incorrect, pls correct me!

If you like my answer and explanation, mark me as brainliest!

-Chetan K

Answer: 301.6

-------------------

answer above isn't right this is, trust me.

joey is going shopping for a new pair of sneakers. He finds a pair that have an original price of $155. They are on sale today for 30% off. How much does Joey pay for the sneakers including 8% sales tax?

Answers

Answer:, Joey will pay $117.18 for sneakers.

Step-by-step explanation:

Given: original price = $155

Discount rate = 30%

Tax rate = 8%

Price after discount = Original price - (Discount) x (original price)

[tex]= 155-0.30\times 155\\\\=155-46.5\\\\=\$\ 108.5[/tex]

Tax = Tax rate x (Price after discount)

[tex]= 0.08 \times 108.5[/tex]

= $ 8.68

Final price for sneakers = Price after discount + Tax

= $ (108.5+8.68)

= $ 117.18

Hence

Find the volume of the solid. Round your answer to the nearest hundredth.

Answers

Answer: 27.6

Step-by-step explanation:

tan 36 = 1/x

x =1/ tan 36 x = 1.38 which is your apothem

area of pentagon = 5x2x1.38/2 = 6.9

now do 6.9 x 4 = 27.6

The volume of the solid is 27.52 cubic yards

What is Three dimensional shape?

a three dimensional shape can be defined as a solid figure or an object or shape that has three dimensions—length, width, and height.

The given figure is a pentagonal prism

The base length of prism is 2yd

The height of prism is 4 yd

Volume of pentagonal prism V = 1/4 √(5(5+2√5))a²h

a is the base length and h is height

V = 1/4 √(5(5+2√5))4×4

V=27.52

Hence, the volume of the solid is 27.52 cubic yards

To learn more on Three dimensional figure click:

https://brainly.com/question/2400003

#SPJ2

In this addition problem, each letter stands for a different digit.
A
A
+ B B
-------------------
C C C

Find the value of A + B + C?

Answers

The answer is a+b+c equals abc
Other Questions
pls help 20 pointsWhich statements describe the county commission form of government? Check all that apply.Elected commissioners govern the county.Commissioners manage the county budget.An appointed executive administers the county.Commissioners create new county laws.Elected councilors adopt the county budget.An elected executive creates new county laws. 27. Find the length of the segment LK. (G.CO.C.10)Show your work! (2pts)J13xLKX + 6Click to add speaker notes A cat with a mass of 5.00 kg pushes on a 25.0 kg desk with a force of 50.0N to jump off. What is the force on the desk? Several families are planning a shared car trip on scenic drives in New Hampshire's White Mountains. To minimize the possibility of any quarrels, they want to assign individuals to cars so that no two members of a family are in the same car. Explain how to formulate this problem as a network flow problem. After the narrator describes the soldiers backgrounds. Do you believe they are all battle hardened veterans willing to die for the case ?answer watching the video. https://youtu.be/89_3DgW_7mg ? which statement would a proponent of self-determination or black nationalism most likely agree with? A Maria y Juan les _______________ la paellaIs it...1. fascina2. facinanWhich one is it? Please help 2) Jai vu Paul en ville answer in COD I need it urgently Jamal puts $100 in an account that does not earn any interest. Every month after that, he deposits the same amount of money. This sequence represents his account balance for the first few months. $100, $125, $150, What is the explicit formula in function form for the amount of money in his account at the beginning of month n? Cullumber Corporation has announced that its net income for the year ended June 30, 2017, was $1,353,412. The company had EBITDA of $4,948,000, and its depreciation and amortization expense was equal to $1,128,000. The company's average tax rate is 34 percent. What was its interest expense A rock is thrown from the edge of the top of a 51 m tall building at some unknown angle above the horizontal. The rock strikes the ground a horizontal distance of 74 m from the base of the building 8 s after being thrown. Assume that the ground is level and that the side of the building is vertical. Determine the speed with which the rock was thrown. Match each word with the phrase that best defines it. comprehensive centralize coup d'tat infrastructure lyce Complete and wide-ranging A sudden overthrow of an existing government A secondary school that prepares students for UniversityTo bring under the control of a single authority The necessary structures that a country needs to function properly During 2004, Thor Lab supplied hospitals with a comprehensive diagnostic kit for $120. At a volume of 80,000 kits, Thor had fixed costs of $1,000,000 and a profit before income taxes of $200,000. Due to an adverse legal decision, Thors 2005 liability insurance increased by $1,200,000 over 2004. Assuming the volume and other costs are unchanged, what should the 2005 price be if Thor is to make the same $200,000 profit before income taxes?a. $120.00b. $135.00c. $150.00d. $240.00 What is the difference of the polynomials?(5x^3 + 4x^2) - (6x^2 - 2x - 9)Answers:A.) -x^3 + 6x^2 + 9B.) -x^3 + 2x^2 - 9C.) 5x^3 - 2x^2 - 2x - 9D.) 5x^3 - 2x^2 + 2x + 9 x+5y=20x+3y=14 Hello what is this? I need help quick pleasee who nominates the chairperson and vice chairperson to the federal reserve which of the following is the best example of using language to empower someone Why was President Eisenhower so popular? what creates an ionic bond?a: The attraction between neutral ionsb: The checks between similarly charged ionsc: the attraction between oppositely charged ions d: the attraction between neutral atoms